Find a Basis for the solution set

  • Thread starter Thread starter digitol87
  • Start date Start date
  • Tags Tags
    Basis Set
Click For Summary
The discussion revolves around solving a system of linear equations represented in matrix form. The equations are dependent, with the second and third equations being multiples of the first, leading to a reduced row echelon form that reveals only one unique equation. The solution can be expressed in terms of two free variables, indicating that the solution set can be represented as a linear combination of two linearly independent vectors. The user is guided to express x1 in terms of x2 and x3, confirming the relationships among the variables. This approach clarifies the basis for the solution set, emphasizing the dependence of the equations.
digitol87
Messages
3
Reaction score
0
3x1 + x2 + x3 = 0
6x1 + 2x2 + 2x3 = 0
-9x1 - 3x2 - 3x3 = 0


I'm not sure how to approach this problem. I've rewritten these equations as a matrix

[3 1 1]
[6 2 2]
[-9 -3 -3]

Reduced Echelon from gave me this
[3 1 1]
[0 0 0]
[0 0 0]

Am I approaching this the wrong way?
What should I do next? Please help.
Thank You.
 
Last edited:
Physics news on Phys.org
digitol87 said:
3x1 + x2 + x3 = 0
6x1 + 2x2 + 2x3 = 0
-9x1 - 3x2 - 3x3 = 0
By inspection it can be seen that the 2nd equation is 2 times the first, and the 3rd is -3 times the first. In essence, you have the same equation written three times.
digitol87 said:
I'm not sure how to approach this problem. I've rewritten these equations as a matrix

[3 1 1]
[6 2 2]
[-9 -3 -3]

Reduced Echelon from gave me this
[3 1 1]
[0 0 0]
[0 0 0]

Am I approaching this the wrong way?
What should I do next? Please help.
Thank You.

Solve the first equation to get
x1 = -(1/3)x2 - (1/3)x3
x2 = x2 + 0x3
x3 = 0x2 + x3

The 2nd and 3rd equations above are obviously true.

If you stare at this system awhile, you might see that any vector <x1, x2, x3> in this set can be written as a linear combination of two vectors that happen to be linearly independent.
 
Question: A clock's minute hand has length 4 and its hour hand has length 3. What is the distance between the tips at the moment when it is increasing most rapidly?(Putnam Exam Question) Answer: Making assumption that both the hands moves at constant angular velocities, the answer is ## \sqrt{7} .## But don't you think this assumption is somewhat doubtful and wrong?

Similar threads

  • · Replies 24 ·
Replies
24
Views
3K
  • · Replies 14 ·
Replies
14
Views
3K
  • · Replies 9 ·
Replies
9
Views
3K
  • · Replies 4 ·
Replies
4
Views
2K
Replies
4
Views
2K
  • · Replies 2 ·
Replies
2
Views
2K
Replies
6
Views
2K
  • · Replies 7 ·
Replies
7
Views
2K
Replies
3
Views
2K
  • · Replies 5 ·
Replies
5
Views
4K